Is this sequence increasing or decreasing? Is it bounded? How do I more formally show the bound? The Next CEO of Stack OverflowThe average of a bounded, decreasing-difference sequencePointwise limit,$f$, of the sequence is not boundedMonotone non-decreasing sequence with upper boundShowing that $a_n = 1-(frac-23)^n$ is not boundedHow do I show this sequence of functions is uniformly bounded?Is sequence $7a_n+1 = a_n^2+3$ bounded, increasing? And find its limit as $nto+infty$Finding a sum to infinity with a factorialHow to prove a sequence is increasingHow to prove a sequence is bounded above or belowShow that $x_n+2 = x_n+1 + fracx_n2^n$ is a bounded sequence

Is it convenient to ask the journal's editor for two additional days to complete a review?

What day is it again?

How did Beeri the Hittite come up with naming his daughter Yehudit?

Can you teleport closer to a creature you are Frightened of?

Can I board the first leg of the flight without having final country's visa?

Can Sneak Attack be used when hitting with an improvised weapon?

Do I need to write [sic] when including a quotation with a number less than 10 that isn't written out?

(How) Could a medieval fantasy world survive a magic-induced "nuclear winter"?

Can I calculate next year's exemptions based on this year's refund/amount owed?

Towers in the ocean; How deep can they be built?

What steps are necessary to read a Modern SSD in Medieval Europe?

What is the difference between "hamstring tendon" and "common hamstring tendon"?

Physiological effects of huge anime eyes

Does higher Oxidation/ reduction potential translate to higher energy storage in battery?

Are the names of these months realistic?

Is it okay to majorly distort historical facts while writing a fiction story?

Would a grinding machine be a simple and workable propulsion system for an interplanetary spacecraft?

Is it ok to trim down a tube patch?

Defamation due to breach of confidentiality

Is a distribution that is normal, but highly skewed, considered Gaussian?

Computationally populating tables with probability data

What difference does it make using sed with/without whitespaces?

Is it professional to write unrelated content in an almost-empty email?

Redefining symbol midway through a document



Is this sequence increasing or decreasing? Is it bounded? How do I more formally show the bound?



The Next CEO of Stack OverflowThe average of a bounded, decreasing-difference sequencePointwise limit,$f$, of the sequence is not boundedMonotone non-decreasing sequence with upper boundShowing that $a_n = 1-(frac-23)^n$ is not boundedHow do I show this sequence of functions is uniformly bounded?Is sequence $7a_n+1 = a_n^2+3$ bounded, increasing? And find its limit as $nto+infty$Finding a sum to infinity with a factorialHow to prove a sequence is increasingHow to prove a sequence is bounded above or belowShow that $x_n+2 = x_n+1 + fracx_n2^n$ is a bounded sequence










0












$begingroup$


So I have this general equation for a sequence:



$$a_n = frac1-n2+n$$



is it increasing or decraesing? Writing out the first few terms I have:



$$0, frac-14, frac-25,frac-36,frac-47,frac-58$$



using a derivative test, we get:



$$f(x)= frac1-x2+x$$



$$f'(x)= -1$$



so the sequence is decraesing as $n -> infty$



Since the first term is 0, it's bounded above by 0. And it looks like it never crosses -1 so it's bounded below by -1. But how do I show this more vigorously?



Taking the limits seems like half the answer. What if the function oscillates around the limit but cross it? Taking the limit doesn't seem like the complete way to find the bounds right?










share|cite|improve this question









$endgroup$











  • $begingroup$
    Show $a_n+1-a_n < 0$.
    $endgroup$
    – amsmath
    Mar 28 at 1:04















0












$begingroup$


So I have this general equation for a sequence:



$$a_n = frac1-n2+n$$



is it increasing or decraesing? Writing out the first few terms I have:



$$0, frac-14, frac-25,frac-36,frac-47,frac-58$$



using a derivative test, we get:



$$f(x)= frac1-x2+x$$



$$f'(x)= -1$$



so the sequence is decraesing as $n -> infty$



Since the first term is 0, it's bounded above by 0. And it looks like it never crosses -1 so it's bounded below by -1. But how do I show this more vigorously?



Taking the limits seems like half the answer. What if the function oscillates around the limit but cross it? Taking the limit doesn't seem like the complete way to find the bounds right?










share|cite|improve this question









$endgroup$











  • $begingroup$
    Show $a_n+1-a_n < 0$.
    $endgroup$
    – amsmath
    Mar 28 at 1:04













0












0








0





$begingroup$


So I have this general equation for a sequence:



$$a_n = frac1-n2+n$$



is it increasing or decraesing? Writing out the first few terms I have:



$$0, frac-14, frac-25,frac-36,frac-47,frac-58$$



using a derivative test, we get:



$$f(x)= frac1-x2+x$$



$$f'(x)= -1$$



so the sequence is decraesing as $n -> infty$



Since the first term is 0, it's bounded above by 0. And it looks like it never crosses -1 so it's bounded below by -1. But how do I show this more vigorously?



Taking the limits seems like half the answer. What if the function oscillates around the limit but cross it? Taking the limit doesn't seem like the complete way to find the bounds right?










share|cite|improve this question









$endgroup$




So I have this general equation for a sequence:



$$a_n = frac1-n2+n$$



is it increasing or decraesing? Writing out the first few terms I have:



$$0, frac-14, frac-25,frac-36,frac-47,frac-58$$



using a derivative test, we get:



$$f(x)= frac1-x2+x$$



$$f'(x)= -1$$



so the sequence is decraesing as $n -> infty$



Since the first term is 0, it's bounded above by 0. And it looks like it never crosses -1 so it's bounded below by -1. But how do I show this more vigorously?



Taking the limits seems like half the answer. What if the function oscillates around the limit but cross it? Taking the limit doesn't seem like the complete way to find the bounds right?







sequences-and-series






share|cite|improve this question













share|cite|improve this question











share|cite|improve this question




share|cite|improve this question










asked Mar 28 at 1:01









Jwan622Jwan622

2,33411632




2,33411632











  • $begingroup$
    Show $a_n+1-a_n < 0$.
    $endgroup$
    – amsmath
    Mar 28 at 1:04
















  • $begingroup$
    Show $a_n+1-a_n < 0$.
    $endgroup$
    – amsmath
    Mar 28 at 1:04















$begingroup$
Show $a_n+1-a_n < 0$.
$endgroup$
– amsmath
Mar 28 at 1:04




$begingroup$
Show $a_n+1-a_n < 0$.
$endgroup$
– amsmath
Mar 28 at 1:04










2 Answers
2






active

oldest

votes


















2












$begingroup$

Hint: Write it as $a_n=-1+frac 32+n$ and it is clearer.






share|cite|improve this answer









$endgroup$




















    2












    $begingroup$

    Note that



    $$a_n = frac1-n2+n = frac-2-n+32+n = -1 + frac32+n tag1labeleq1$$



    Thus, for $n gt -2$, $a_n gt -1$. Also, this allows you to determine that $lim_n to inftya_n = -1$ since $frac32+n$ goes to $0$.






    share|cite|improve this answer









    $endgroup$













      Your Answer





      StackExchange.ifUsing("editor", function ()
      return StackExchange.using("mathjaxEditing", function ()
      StackExchange.MarkdownEditor.creationCallbacks.add(function (editor, postfix)
      StackExchange.mathjaxEditing.prepareWmdForMathJax(editor, postfix, [["$", "$"], ["\\(","\\)"]]);
      );
      );
      , "mathjax-editing");

      StackExchange.ready(function()
      var channelOptions =
      tags: "".split(" "),
      id: "69"
      ;
      initTagRenderer("".split(" "), "".split(" "), channelOptions);

      StackExchange.using("externalEditor", function()
      // Have to fire editor after snippets, if snippets enabled
      if (StackExchange.settings.snippets.snippetsEnabled)
      StackExchange.using("snippets", function()
      createEditor();
      );

      else
      createEditor();

      );

      function createEditor()
      StackExchange.prepareEditor(
      heartbeatType: 'answer',
      autoActivateHeartbeat: false,
      convertImagesToLinks: true,
      noModals: true,
      showLowRepImageUploadWarning: true,
      reputationToPostImages: 10,
      bindNavPrevention: true,
      postfix: "",
      imageUploader:
      brandingHtml: "Powered by u003ca class="icon-imgur-white" href="https://imgur.com/"u003eu003c/au003e",
      contentPolicyHtml: "User contributions licensed under u003ca href="https://creativecommons.org/licenses/by-sa/3.0/"u003ecc by-sa 3.0 with attribution requiredu003c/au003e u003ca href="https://stackoverflow.com/legal/content-policy"u003e(content policy)u003c/au003e",
      allowUrls: true
      ,
      noCode: true, onDemand: true,
      discardSelector: ".discard-answer"
      ,immediatelyShowMarkdownHelp:true
      );



      );













      draft saved

      draft discarded


















      StackExchange.ready(
      function ()
      StackExchange.openid.initPostLogin('.new-post-login', 'https%3a%2f%2fmath.stackexchange.com%2fquestions%2f3165339%2fis-this-sequence-increasing-or-decreasing-is-it-bounded-how-do-i-more-formally%23new-answer', 'question_page');

      );

      Post as a guest















      Required, but never shown

























      2 Answers
      2






      active

      oldest

      votes








      2 Answers
      2






      active

      oldest

      votes









      active

      oldest

      votes






      active

      oldest

      votes









      2












      $begingroup$

      Hint: Write it as $a_n=-1+frac 32+n$ and it is clearer.






      share|cite|improve this answer









      $endgroup$

















        2












        $begingroup$

        Hint: Write it as $a_n=-1+frac 32+n$ and it is clearer.






        share|cite|improve this answer









        $endgroup$















          2












          2








          2





          $begingroup$

          Hint: Write it as $a_n=-1+frac 32+n$ and it is clearer.






          share|cite|improve this answer









          $endgroup$



          Hint: Write it as $a_n=-1+frac 32+n$ and it is clearer.







          share|cite|improve this answer












          share|cite|improve this answer



          share|cite|improve this answer










          answered Mar 28 at 1:10









          Ross MillikanRoss Millikan

          300k24200375




          300k24200375





















              2












              $begingroup$

              Note that



              $$a_n = frac1-n2+n = frac-2-n+32+n = -1 + frac32+n tag1labeleq1$$



              Thus, for $n gt -2$, $a_n gt -1$. Also, this allows you to determine that $lim_n to inftya_n = -1$ since $frac32+n$ goes to $0$.






              share|cite|improve this answer









              $endgroup$

















                2












                $begingroup$

                Note that



                $$a_n = frac1-n2+n = frac-2-n+32+n = -1 + frac32+n tag1labeleq1$$



                Thus, for $n gt -2$, $a_n gt -1$. Also, this allows you to determine that $lim_n to inftya_n = -1$ since $frac32+n$ goes to $0$.






                share|cite|improve this answer









                $endgroup$















                  2












                  2








                  2





                  $begingroup$

                  Note that



                  $$a_n = frac1-n2+n = frac-2-n+32+n = -1 + frac32+n tag1labeleq1$$



                  Thus, for $n gt -2$, $a_n gt -1$. Also, this allows you to determine that $lim_n to inftya_n = -1$ since $frac32+n$ goes to $0$.






                  share|cite|improve this answer









                  $endgroup$



                  Note that



                  $$a_n = frac1-n2+n = frac-2-n+32+n = -1 + frac32+n tag1labeleq1$$



                  Thus, for $n gt -2$, $a_n gt -1$. Also, this allows you to determine that $lim_n to inftya_n = -1$ since $frac32+n$ goes to $0$.







                  share|cite|improve this answer












                  share|cite|improve this answer



                  share|cite|improve this answer










                  answered Mar 28 at 1:10









                  John OmielanJohn Omielan

                  4,4512215




                  4,4512215



























                      draft saved

                      draft discarded
















































                      Thanks for contributing an answer to Mathematics Stack Exchange!


                      • Please be sure to answer the question. Provide details and share your research!

                      But avoid


                      • Asking for help, clarification, or responding to other answers.

                      • Making statements based on opinion; back them up with references or personal experience.

                      Use MathJax to format equations. MathJax reference.


                      To learn more, see our tips on writing great answers.




                      draft saved


                      draft discarded














                      StackExchange.ready(
                      function ()
                      StackExchange.openid.initPostLogin('.new-post-login', 'https%3a%2f%2fmath.stackexchange.com%2fquestions%2f3165339%2fis-this-sequence-increasing-or-decreasing-is-it-bounded-how-do-i-more-formally%23new-answer', 'question_page');

                      );

                      Post as a guest















                      Required, but never shown





















































                      Required, but never shown














                      Required, but never shown












                      Required, but never shown







                      Required, but never shown

































                      Required, but never shown














                      Required, but never shown












                      Required, but never shown







                      Required, but never shown







                      Popular posts from this blog

                      Triangular numbers and gcdProving sum of a set is $0 pmod n$ if $n$ is odd, or $fracn2 pmod n$ if $n$ is even?Is greatest common divisor of two numbers really their smallest linear combination?GCD, LCM RelationshipProve a set of nonnegative integers with greatest common divisor 1 and closed under addition has all but finite many nonnegative integers.all pairs of a and b in an equation containing gcdTriangular Numbers Modulo $k$ - Hit All Values?Understanding the Existence and Uniqueness of the GCDGCD and LCM with logical symbolsThe greatest common divisor of two positive integers less than 100 is equal to 3. Their least common multiple is twelve times one of the integers.Suppose that for all integers $x$, $x|a$ and $x|b$ if and only if $x|c$. Then $c = gcd(a,b)$Which is the gcd of 2 numbers which are multiplied and the result is 600000?

                      Ingelân Ynhâld Etymology | Geografy | Skiednis | Polityk en bestjoer | Ekonomy | Demografy | Kultuer | Klimaat | Sjoch ek | Keppelings om utens | Boarnen, noaten en referinsjes Navigaasjemenuwww.gov.ukOffisjele webside fan it regear fan it Feriene KeninkrykOffisjele webside fan it Britske FerkearsburoNederlânsktalige ynformaasje fan it Britske FerkearsburoOffisjele webside fan English Heritage, de organisaasje dy't him ynset foar it behâld fan it Ingelske kultuergoedYnwennertallen fan alle Britske stêden út 'e folkstelling fan 2011Notes en References, op dizze sideEngland

                      Հադիս Բովանդակություն Անվանում և նշանակություն | Դասակարգում | Աղբյուրներ | Նավարկման ցանկ